Re: [obm-l] Ajuda

2004-07-06 Por tôpico Bernardo Freitas Paulo da Costa
Bom, se você souber derivadas, basta derivar f(x) com relação a x, e igualar a zero, obtendo 0 = f'(x) = 2( (x-1) + (x-2) + (x-3) + ... + (x-50) ) o que reduz-se a soma de P.A: 0 = 50x - (1+2+3+...+50)= 50x - 50*51/2) ou seja, x = 25.5. Como é esperado que x seja inteiro, pelas suas respostas, e

Re: RES: [obm-l] Problema de Divisibilidade / Primos

2004-07-20 Por tôpico Bernardo Freitas Paulo da Costa
Oi, David, Enumere os primos menores do que 20: 2, 3, 5, 7, 11, 13, 17, 19: são 8. Um número que satisfaça as condições do enunciado pode ter, no máximo, um de cada um destes fatores, pela segunda parte, e nenhum outro fator, pela primeira parte. Assim, temos um problema de combinatória, agora:

Re: [obm-l] Re: [obm-l] análise

2004-08-06 Por tôpico Bernardo Freitas Paulo da Costa
Oi, Artur. Eu acho que quando estava escrito |f(x)| era para ser interpretado como, usando a sua notac~ao f=(f1, f2, ,..., fn) (f1^2 + f2^2 + ... + fn^2)^(1/2). A'i eu acho que a an'alise da quest~ao 'e mais complicada, mas (se eu n~ao me engano, estudei isso h'a muito tempo atr'as) deve

Re: [obm-l] pergunta do aluno

2004-08-10 Por tôpico Bernardo Freitas Paulo da Costa
Perceba que C(n, n-k) = C(n,k), logo podemos utilizar a relação de Stiefel C(n-1, k-1) + C(n-1, k) = C(n,k) para obter C(n-1,k) = 60 - 18 = 42. Abraços, Bernardo On Tue, 10 Aug 2004, nilton rr wrote: Companheiros essa pergunta foi feita por um dos meus alunos ,peço ajuda pois não consegui

Re: [obm-l] questao simples do bartle

2004-08-11 Por tôpico Bernardo Freitas Paulo da Costa
elementos de B (Tome como base que A^{x} tem, obviamente, #A elementos). Abraços, Bernardo Freitas Paulo da Costa On Tue, 10 Aug 2004, niski wrote: Pessoal, este problema tirado do capitulo 8 (The Topology of Cartesian Spaces) me parece ser simples por ser um dos primeiros do capitulo. Eu realmente

Re: [obm-l] Re:[obm-l] Problema envolvendo potências

2004-09-10 Por tôpico Bernardo Freitas Paulo da Costa
Bom, a idéia que você teve está quase certa, mas você deslizou na hora de fazer a divisão (pois aí o sinal da desigualdade muda). Temos, como você falou, x = (3^31 + 2^31)/(3^29 + 2^29) = (9 + 4y)/(1 + y), onde y = (2/3)^29 Podemos escrever 9 + 4y = 9 + 9y - 5y = 9(1 + y) - 5y, e

Re: [obm-l] Conjuntos enumeraveis e finitos

2004-09-13 Por tôpico Bernardo Freitas Paulo da Costa
para entrar na lista, sair da lista e usar a lista em http://www.mat.puc-rio.br/~nicolau/olimp/obm-l.html = -- Bernardo Freitas Paulo da Costa

Re: [obm-l] Re: [obm-l] Questão_4

2004-09-14 Por tôpico Bernardo Freitas Paulo da Costa
, = Instruções para entrar na lista, sair da lista e usar a lista em http://www.mat.puc-rio.br/~nicolau/olimp/obm-l.html = -- Bernardo Freitas Paulo da

Re: [obm-l] limites iterados

2004-09-19 Por tôpico Bernardo Freitas Paulo da Costa
Bom, eu acho que você entendeu o suficiente de limite para ter a idéia certa, mas apenas faltou traduzir o desenho em epsilons e deltas. Uma idéia é a seguinte: você quer provar que lim y-b g(y) = L, então basta calcular a diferença g(y) - L e ver se ela vai dar pequena o suficiente para y

Re: [obm-l] Conjunto dos algébricos

2004-09-24 Por tôpico Bernardo Freitas Paulo da Costa
/~nicolau/olimp/obm-l.html = -- Bernardo Freitas Paulo da Costa = Instruções para entrar na lista, sair da lista e usar a lista em http://www.mat.puc

Re: [obm-l] Parcelas de 1998

2004-10-15 Por tôpico Bernardo Freitas Paulo da Costa
= [2^(beta)]*[3^(gama)] com (beta = 1 ou 2). Como 1998 = 3*666 + 0, P = 3^666 e S = 3 + 3 + 3 + 3 +...+ 3 (666 vezes) -- Bernardo Freitas Paulo da Costa = Instrues para entrar na lista, sair da lista e usar a lista

Re: [obm-l] Desigualdade das Médias

2004-10-04 Por tôpico Bernardo Freitas Paulo da Costa
Bom, eu acho em particular que você nunca vai precisar demostrar a desigualdade das médias numa prova de olimpíada. Quanto ao caso de vestibulares, talvez seja bom você citar o teorema com algo do tipo Sabemos que MH = MG = MA. Elas são as médias Harmônica, Geométrica e Aritmética,

Re: [obm-l] Equações trigonométricas [3 problemas]

2004-10-07 Por tôpico Bernardo Freitas Paulo da Costa
produtos, bem como arco duplo, etc. Na segunda, o truque é usar números complexos e somar as duas P.Gs que vão aparecer quando você escrever os cossenos. A terceira já responderam. -- Bernardo Freitas Paulo da Costa On Wed, 6 Oct 2004 12:07:53 -0700 (PDT), Felipe Torres [EMAIL PROTECTED] wrote: Olá

Re: [obm-l] 1000 primeiros dígitos de n^1998

2004-10-18 Por tôpico Bernardo Freitas Paulo da Costa
PROTECTED] at [EMAIL PROTECTED] wrote: Olá pessoal ! Prove que existe n pertencente a N tal que os 1000 primeiros dígitos de n^1998 são iguais a 1. -- Bernardo Freitas Paulo da Costa

Re: [obm-l] resto

2004-10-20 Por tôpico Bernardo Freitas Paulo da Costa
que abrir o binômio??. Valeu, Korshinói -- Bernardo Freitas Paulo da Costa = Instruções para entrar na lista, sair da lista e usar a lista em http://www.mat.puc-rio.br/~nicolau/olimp/obm-l.html =

Re: [obm-l] Re: [obm-l] OBM2004 - NIVEL U - Problem a 2 - Uma variação

2004-10-20 Por tôpico Bernardo Freitas Paulo da Costa
= -- Bernardo Freitas Paulo da Costa = Instruções para entrar na lista, sair da lista e usar a lista em http://www.mat.puc-rio.br/~nicolau/olimp/obm-l.html =

Re: [obm-l] Livro de Topologia geral

2004-10-29 Por tôpico Bernardo Freitas Paulo da Costa
://www.mat.puc-rio.br/~nicolau/olimp/obm-l.html = -- Bernardo Freitas Paulo da Costa = Instruções para entrar na lista, sair da lista e usar a lista em http

Re: [obm-l] Medias e Divisores

2004-10-29 Por tôpico Bernardo Freitas Paulo da Costa
. = Instruções para entrar na lista, sair da lista e usar a lista em http://www.mat.puc-rio.br/~nicolau/olimp/obm-l.html = -- Bernardo Freitas Paulo da Costa

Re: [obm-l] [OFF-TOPIC] Universidades - Curso de Matemática

2004-10-29 Por tôpico Bernardo Freitas Paulo da Costa
para entrar na lista, sair da lista e usar a lista em http://www.mat.puc-rio.br/~nicolau/olimp/obm-l.html = -- Bernardo Freitas Paulo da Costa

Re: [obm-l] Limite e continuidade

2004-11-07 Por tôpico Bernardo Freitas Paulo da Costa
entrar na lista, sair da lista e usar a lista em http://www.mat.puc-rio.br/~nicolau/olimp/obm-l.html = -- Bernardo Freitas Paulo da Costa

Re: [obm-l] ALGEBRA DE BOOLE!

2004-11-17 Por tôpico Bernardo Freitas Paulo da Costa
= = Instruções para entrar na lista, sair da lista e usar a lista em http://www.mat.puc-rio.br/~nicolau/olimp/obm-l.html = -- Bernardo Freitas Paulo da Costa

Re: [obm-l] Método Numérico

2004-11-24 Por tôpico Bernardo Freitas Paulo da Costa
Oi, O Livro do Elon Análise Real apresenta um tópico sobre Método de Newton (eu acho que é na seção de aplicações da derivada) que explica muito bem porquê é quadrático, quais as hipóteses necessárias, etc. Vale a pena ver para ter uma orientação. Abraços, -- Bernardo Freitas Paulo da Costa

Re: [obm-l] Duvidas

2004-11-25 Por tôpico Bernardo Freitas Paulo da Costa
= -- Bernardo Freitas Paulo da Costa = Instruções para entrar na lista, sair da lista e usar a lista em http://www.mat.puc-rio.br/~nicolau/olimp/obm-l.html =

Re: [obm-l] Duvidas

2004-11-25 Por tôpico Bernardo Freitas Paulo da Costa
:58:17 -0200, Bernardo Freitas Paulo da Costa [EMAIL PROTECTED] wrote: Tem mínimo sim: Usando que mdc(a, b) = mdc(a, b-a) = mdc(a-b, b), temos, sucessivamente: mdc(2n + 4, 4n + 2) = mdc(2n + 4, 2n - 2) = mdc(6, 2n - 2) = 6 Abraços, Bernardo Costa On Thu, 25 Nov 2004 05:39:53 -0300, Fernando

Re: [obm-l] sistema linear

2004-12-01 Por tôpico Bernardo Freitas Paulo da Costa
soluo do problema. Ento, basta tomar as combinaes lineares dos mesmos (que formam um plano, como voc disse). Esse um dos problemas da RPM que mais me convence que lgebra Linear importantssimo. Mesmo que PAREA uma questo que d para resolver no brao. -- Bernardo Freitas Paulo da Costa On Tue, 30 Nov

Re: [obm-l] Perguntas simples para respostas convincentes

2004-12-09 Por tôpico Bernardo Freitas Paulo da Costa
Oi, Acho que o lastro-ouro já acabou há tempos. Talvez com a excessão da Alemanha e outros poucos países, o que existe é que cada país possui uma cesta de moedas, como por exemplo dólar, yen, libra, etc... que definem quanto vale a moeda local. Abraços, -- Bernardo Freitas Paulo da Costa

Re: [obm-l] Questão de função

2004-12-09 Por tôpico Bernardo Freitas Paulo da Costa
(a^(x - y) + a^(-x + y) ) = 1/2( a^x a^(-y) + a^(-x) a^y) Multiplicando f(x)f(y) temos: 1/4 (a^x + a^(-x))(a^y + a^(-y)) = 1/4 (a^x a^y + a^x a^(-y) + a^(-x) a^y + a^(-x) a^(-y) ). Multiplique por dois e confira que o resultado é a soma f(x + y) + f(x - y). Abraços, -- Bernardo Freitas Paulo da

Re: [obm-l] PARTE INTEIRA

2004-12-17 Por tôpico Bernardo Freitas Paulo da Costa
ArrZero(-2,1) = -2. Esta relação, entretanto, não é tão útil como as outras duas... Abraços, -- Bernardo Freitas Paulo da Costa On Thu, 16 Dec 2004 22:20:46 -0300 (ART), Jorge Paulino [EMAIL PROTECTED] wrote: A parte inteira de um número positivo não gera equívoco. Por exemplo, a parte inteira de

Re: [obm-l] PARTE INTEIRA

2004-12-19 Por tôpico Bernardo Freitas Paulo da Costa
uma questão que falava sobre parte inteira, onde vinha exatamente a definição acima, junto com os exemplos: [2] = 2, [pi] = 3, [-pi] = -4 (ou alguma coisa assim). Abraços, -- Bernardo Freitas Paulo da Costa On Sat, 18 Dec 2004 22:14:27 -0300 (ART), [EMAIL PROTECTED] [EMAIL PROTECTED] wrote: Olá

Re: [obm-l] 1 -1/2 +1/3.......= Ln(2)

2004-12-21 Por tôpico Bernardo Freitas Paulo da Costa
garantir que a série converge no limite r- 1 também (este passo é um pouco mais difícil do que parece: tente provar, vale a pena!. Integre até 1-eps e faça eps-0, veja que você pode fazer isso e obtenha o resultado), e portanto está provado. Qualquer dúvidas, fale. -- Bernardo Freitas Paulo da Costa

Re: [obm-l] Probleminha....

2004-12-28 Por tôpico Bernardo Freitas Paulo da Costa
e bom Ano-novo, -- Bernardo Freitas Paulo da Costa On Tue, 28 Dec 2004 17:11:19 -0200, Osvaldo Mello Sponquiado [EMAIL PROTECTED] wrote: Eu usei calculo, tambem acaba sendo simples. Eh facil mostrar que so precisamos nos deter no conjunto (0,1/e) x (0,1/e). Para isto, observamos que se 0

Re: [obm-l] Probleminha....

2004-12-31 Por tôpico Bernardo Freitas Paulo da Costa
a seguir: 1) f(x)^g(x) não existe (use algo patológico como sen(1/x), sempre funciona...) 2) f(x)^g(x) = r para um real r arbitrário (bom, pode ser complexo também, se você quiser...) 3) f(x)^g(x) diverge para +- infinito Bom, sem mais, Bom ano novo a todos da lista, -- Bernardo Freitas Paulo da Costa

Re: [obm-l] Cardinalidade

2005-01-06 Por tôpico Bernardo Freitas Paulo da Costa
Oi, A solução do Domingos usa o axioma da escolha? Onde? Abraços, -- Bernardo Freitas Paulo da Costa On Thu, 6 Jan 2005 15:32:32 -0200, Nicolau C. Saldanha [EMAIL PROTECTED] wrote: On Thu, Jan 06, 2005 at 02:08:21PM -0200, Artur Costa Steiner wrote: Boa tarde, Eu ainda nao consegui

Re: [obm-l] IME X ITA

2005-01-13 Por tôpico Bernardo Freitas Paulo da Costa
O link da CAPES foi errado, é http://www.capes.gov.br/Documentos/Avaliacao2004/AvTrienal2004_FinalPorArea.pdf -- Bernardo Freitas Paulo da Costa = Instruções para entrar na lista, sair da lista e usar a lista em http

Re: [obm-l] IME X ITA

2005-01-13 Por tôpico Bernardo Freitas Paulo da Costa
mais informações eu estarei à disposição. Espero que tenha ajudado (e botar lenha na fogueira é sempre bom!) -- Bernardo Freitas Paulo da Costa On Tue, 11 Jan 2005 23:26:46 -0200, Bruno Bruno [EMAIL PROTECTED] wrote: vocês falam muito do ime e do ita, mas e qto a ufrj? ouvi dizer que eles tem

Re: [obm-l] Re: Sequencias

2005-01-21 Por tôpico Bernardo Freitas Paulo da Costa
= -- Bernardo Freitas Paulo da Costa = Instruções para entrar na lista, sair da lista e usar a lista em http://www.mat.puc-rio.br/~nicolau/olimp/obm-l.html =

Re: [obm-l] eq diofantinas

2005-02-02 Por tôpico Bernardo Freitas Paulo da Costa
e aumentar o outro. Abraços, -- Bernardo Freitas Paulo da Costa On Wed, 2 Feb 2005 12:19:34 -0300 (ART), Marcelo Ribeiro [EMAIL PROTECTED] wrote: 1) Eu não entendi o porquê da restrição c=ab... Bom, seja d = mdc(a,b). É possível escrever d como combinação linear dos números a e b, isto é

Re: [obm-l] alg. linear - oper. normal (problema)

2005-02-14 Por tôpico Bernardo Freitas Paulo da Costa
Se você souber alguma coisa sobre fechamento, acabou, pois X^t^t = fecho(span(X)), e como Im T e Im T* são espaços vetoriais (ou seja, span(Im T) = Im T ...). Sem pensar muito, acho que é isso. -- Bernardo Freitas Paulo da Costa On Mon, 14 Feb 2005 07:19:14 -0300 (ART), Lista OBM [EMAIL

Re: [obm-l] Teorema de Gauss

2005-02-17 Por tôpico Bernardo Freitas Paulo da Costa
, dá para demonstrar. Abraços, -- Bernardo Freitas Paulo da Costa On Wed, 16 Feb 2005 20:17:35 -0300 (ART), Alan Pellejero [EMAIL PROTECTED] wrote: Olá pessoal, alguém poderia me dar uma dica de como eu posso fazer a demonstração para alunos do ensino médio do teorema de gauss que trata sobre

Re: [obm-l] LIMITAÇÕES TECNOLÓGICAS!

2005-02-18 Por tôpico Bernardo Freitas Paulo da Costa
Bom, se você quiser multiplicar pi por e eu não vejo como fazer isso só com o botão de adição... -- Bernardo Freitas Paulo da Costa On Thu, 17 Feb 2005 16:59:38 -0500, Qwert Smith [EMAIL PROTECTED] wrote: Depende da calculadora que vc tem... vc sempre pode gastar seu dedo no botao da adicao

Re: [obm-l] Matriz

2005-02-19 Por tôpico Bernardo Freitas Paulo da Costa
matrizes quaisquer, provar que AB não é invertível. Abraços Vinícius Meireles Aleixo -- Bernardo Freitas Paulo da Costa = Instruções para entrar na lista, sair da lista e usar a lista em http://www.mat.puc-rio.br

Re: [obm-l] Sugestão sobre cursos em julho ( OFF-LINE)

2005-02-22 Por tôpico Bernardo Freitas Paulo da Costa
Freitas Paulo da Costa On Tue, 22 Feb 2005 11:44:17 -0300 (ART), Alan Pellejero [EMAIL PROTECTED] wrote: Olá pessoal! Gostaria de participar de algum curso em julho e gostaria de receber sugestões sobre cursos. Interesso-me pela ufpr, uel, usp e impa. Um grande abraço e desculpe pelo off-topic

Re: [obm-l] sexta feira 13...

2005-02-23 Por tôpico Bernardo Freitas Paulo da Costa
só um, ou seja, não é tanto tempo para azar... - temos a solução.) Abraços, -- Bernardo Freitas Paulo da Costa On Wed, 23 Feb 2005 08:27:49 -0300, carlos gomes [EMAIL PROTECTED] wrote: Pessoal, E verdade que todo ano tem pelo menos uma sexta-feira 13? Se for verdade como verifico isto

Re: [obm-l] RE:[off topic]Química Correção

2005-03-03 Por tôpico Bernardo Freitas Paulo da Costa
He-4 para retirar a influência dos elétrons (o que é mais fácil). Abraços, -- Bernardo Freitas Paulo da Costa On Thu, 3 Mar 2005 15:37:04 -0300 (ART), Charles Quevedo [EMAIL PROTECTED] wrote: Li ontem meus e-mails da lista e percebi entre as centenas de mensagens acumuladas uma cujo assunto éra

Re: [obm-l] Re: [obm-l] Exercicios difíceis

2005-03-07 Por tôpico Bernardo Freitas Paulo da Costa
-- Bernardo Freitas Paulo da Costa On Sun, 6 Mar 2005 21:46:19 -0300, Brunno [EMAIL PROTECTED] wrote: 1- Durante 40s seguindo para o norte é percorrido 40x300cm com direcao ao norte Durante os 40s percorridos para o leste em movimento uniformemente variado s=(at^2)/2=10x1600/2=10x800cm

Re: [obm-l] C-homogeneidade implica C-Linearidade?

2005-03-16 Por tôpico Bernardo Freitas Paulo da Costa
= Yahoo! Mail - Com 250MB de espaço. Abra sua conta! -- Bernardo Freitas Paulo da Costa = Instruções para entrar na lista

Re: [obm-l] C-homogeneidade implica C-Linearidade?

2005-03-16 Por tôpico Bernardo Freitas Paulo da Costa
Oi, Cláudio. Esta função é exatamente T(z) = z/2 = Re(z) != Im(z) T(a + a*i) = 0, para a = 0 Ou seja, ela é quase T(z) = z/2. Certo? -- Bernardo Freitas Paulo da Costa On Wed, 16 Mar 2005 14:22:44 -0300, claudio.buffara [EMAIL PROTECTED] wrote: Dizer que T(a) = (|a|/2)*u(a), onde u

Re: [obm-l] Proposição

2005-03-18 Por tôpico Bernardo Freitas Paulo da Costa
pode pensar que uma seqüência é a representação binária de um número em [0, 1], mas ainda não sei se é bonitinho... Abraços, -- Bernardo Freitas Paulo da Costa On Fri, 18 Mar 2005 00:25:06 +, [EMAIL PROTECTED] [EMAIL PROTECTED] wrote: Einstein falou uma frase que toca no que você escreveu

Re: [obm-l] ideais maximais

2005-03-18 Por tôpico Bernardo Freitas Paulo da Costa
fazer b(x) = 1 para todo x e obtemos finalmente j = f - ag em J, logo I = C([0,1]) e portanto J é maximal. Acho que é isso. -- Bernardo Freitas Paulo da Costa On Fri, 18 Mar 2005 17:25:18 -0300 (ART), Lista OBM [EMAIL PROTECTED] wrote: Gostaria de uma ajuda no problema abaixo: Seja C([0,1]) o

Re: [obm-l] matemática discreta

2005-03-21 Por tôpico Bernardo Freitas Paulo da Costa
soma. Abraços, -- Bernardo Freitas Paulo da Costa On Mon, 21 Mar 2005 15:05:09 -0300, Thiago Addvico [EMAIL PROTECTED] wrote: olá alguem pode me recomendar um livro bastante completo sobre matemática discreta? não consigo confiar na minha professora então decidi estudar por conta... obrigado

Re: [obm-l] Análise

2005-03-23 Por tôpico Bernardo Freitas Paulo da Costa
diferente de b). Se for outra coisa, avise! -- Bernardo Freitas Paulo da Costa On Tue, 22 Mar 2005 23:09:44 -0300, Diogo [EMAIL PROTECTED] wrote: Pessoal, se puderem me ajudar nesse eu agradeço: Sendo f:[a,b]--R uma função crescente, mostre que, nesse caso, a recíproca do teorema do valor

Re: [obm-l] Re: [obm-l] Análise

2005-03-23 Por tôpico Bernardo Freitas Paulo da Costa
que diz que a derivada de qualquer função tem a PVI; tome agora a função f(x) = x^2 * sen(1/x), cuja derivada é 2x*sen(1/x) - cos(1/x) para x != 0 e f'(0) = 0. Ela tem a PVI, mas não é contínua no zero. Abraços, -- Bernardo Freitas Paulo da Costa On Wed, 23 Mar 2005 09:00:44 -0300

Re: [obm-l] Duvidas

2005-03-30 Por tôpico Bernardo Freitas Paulo da Costa
utilizar ou não uma ferramenta: acho que sim (eu só vi esta solução porque sei congruências!) Abraços, -- Bernardo Freitas Paulo da Costa On Wed, 30 Mar 2005 07:39:00 -0300, matduvidas48 [EMAIL PROTECTED] wrote: Qual é resto da divisão de (99)^2 por 50 ? como resolveria esta

Re: [obm-l] Teo. Riez

2005-03-30 Por tôpico Bernardo Freitas Paulo da Costa
. Abraços -- Bernardo Freitas Paulo da Costa On Wed, 30 Mar 2005 14:34:22 -0300 (ART), Bruno Lima [EMAIL PROTECTED] wrote: Sendo A uma matriz nxn simetrica, positiva definida entao x´Ay (x´ é x transposto ) define um produto interno de x por y . Queria saber se vale a volta: dado um produto

Re: [obm-l] PROBLEMA!

2005-04-01 Por tôpico Bernardo Freitas Paulo da Costa
Note que os maiores somam 15 e os menores, zero. Assim, você já tem alguma coisa. Agora, veja quem pode somar 2 e quem pode somar 13... E depois acho que vale o bom chute. Abraços, -- Bernardo Freitas Paulo da Costa On Apr 1, 2005 12:39 PM, Rafael Alfinito Ferreira [EMAIL PROTECTED] wrote: EU

Re: [obm-l] soma de termos

2005-04-04 Por tôpico Bernardo Freitas Paulo da Costa
*C(n+1, 3) + C(n, 2) (pelo teorema de soma de colunas! - Demonstre que SOMA C(m,k) = C(n+1, k+1) usando a propriedade de que C(a, b) + C(a, b+1) = C(a+1, b+1) ). Agora é só expandir. Abraços, -- Bernardo Freitas Paulo da Costa On Apr 4, 2005 1:07 PM, Brunno [EMAIL PROTECTED] wrote: Boa tarde

Re: [obm-l] soma de termos

2005-04-06 Por tôpico Bernardo Freitas Paulo da Costa
!) Eu acho que vale também para k negativo ou zero, mas isso eu deixo para você pensar (ah, e também tem o velho problema de definir quanto vale C(n, -32), mas isso é zero, eu acho) Para k=0, o teorema na verdade é uma coisa bem trivial! Abraços, -- Bernardo Freitas Paulo da Costa On Apr 4, 2005

Re: [obm-l] Re: [obm-l] Re: [obm-l] cálculo no R^n

2005-04-08 Por tôpico Bernardo Freitas Paulo da Costa
unitário e eps-0. Isso nos dá uma desigualdade acima com 1-eps k 1, para todo eps... então não dá para ser uma contração forte - aquela que tem um k 1 - mas acho que ainda assim o argumento só usa contração fraca) Té mais, -- Bernardo Freitas Paulo da Costa On Apr 8, 2005 1:43 AM, Ronaldo

Re: [obm-l] 1^2 + 2^2 + ... + n^2()Caso Geral

2005-04-11 Por tôpico Bernardo Freitas Paulo da Costa
) (ou algo parecido, pode ter um k-j em vez de j). Daí, como eu falei numa mensagem anterior, é só usar a soma das colunas. Maiores detalhes, você pode encontrar no Concrete Mathematics, R.L.Graham, D.E.Knuth, O.Patashnik. Abraços, -- Bernardo Freitas Paulo da Costa On Apr 9, 2005 11:21 AM

Re: [obm-l] Corpos Redondos(EN)(André!)

2005-04-29 Por tôpico Bernardo Freitas Paulo da Costa
-- Bernardo Freitas Paulo da Costa = Instruções para entrar na lista, sair da lista e usar a lista em http://www.mat.puc-rio.br/~nicolau/olimp/obm-l.html =

Re: [obm-l] Axioma da união

2005-06-16 Por tôpico Bernardo Freitas Paulo da Costa
= {A, B} (acho que pelo axioma do par este C existe ...) e ent~ao voc _define_ A U B como o conjunto dado pelo axioma da uniao aplicado em C. Ate mais, -- Bernardo Freitas Paulo da Costa Os axiomas esto expostos um por um e explicados nas primeiras pginas de Set Theory, de Thomas Jech. []s

Re: [obm-l] numeros binomiais, conjectura

2005-06-18 Por tôpico Bernardo Freitas Paulo da Costa
(para lembrar: n), eles tem que ser iguais! Abraos, -- Bernardo Freitas Paulo da Costa = Instrues para entrar na lista, sair da lista e usar a lista em http://www.mat.puc-rio.br/~nicolau/olimp/obm-l.html =

Re: [obm-l] derivada

2005-06-24 Por tôpico Bernardo Freitas Paulo da Costa
( x* ln(x) ): isso da : ( Derivada de x * ln(x) ) * exp (x * ln(x) ) =repare que chegamos ao mesmo ponto de antes, temos a derivada de g(x) * f(x) = f '(x) E ai é so partir pro abraço. Até mais, -- Bernardo Freitas Paulo da Costa On 6/24/05, Biagio Taffarel [EMAIL PROTECTED] wrote: alguem pode

Re: RES: [obm-l] PA e primos

2005-06-24 Por tôpico Bernardo Freitas Paulo da Costa
me engano) Abraços -- Bernardo Freitas Paulo da Costa On 6/24/05, Artur Costa Steiner [EMAIL PROTECTED] wrote: Isto eh falso (supondo-se uma PA em que os termos sao numeros inteiros). Considere, por exemplo, a PA dos numeros pares, a_n = 2*n, n=1,2,3..Nao eh constante e o unico termo primo eh

Re: RES: [obm-l] Medida

2005-07-06 Por tôpico Bernardo Freitas Paulo da Costa
fato que existe uma sigma-álgebra completa que contém os abertos de R^k para todo k) A x B é mensurável e tem medida zero. Esta demonstraçao está contida na que você deu (bastando notar que B está contido em alguma uniao enumerável dos Q_i). Abraços, -- Bernardo Freitas Paulo da Costa On 7/6/05

Re: [obm-l] Uma desigualdade legal!

2005-07-10 Por tôpico Bernardo Freitas Paulo da Costa
Uma sugestão: ordene a, b e c (por simetria você pode fazer isso). Dai veja que os numeradores e denominadores vão estar ordenados tambem. Dai, use uma desigualdade que tem a ver com ordem... Abraços -- Bernardo Freitas Paulo da Costa On 7/10/05, Marcos Martinelli [EMAIL PROTECTED] wrote: Boa

Re: [obm-l] Re: [obm-l] dúvida conceitual

2005-07-18 Por tôpico Bernardo Freitas Paulo da Costa
leva o seu espaço em R^2: f(a +b, 0, b) = (a, b), que é linear (se você quiser, escreva isso como f(c, 0, b) = (c-b, b) que é claramente linear) T+ -- Bernardo Freitas Paulo da Costa On 7/19/05, Marcos Paulo [EMAIL PROTECTED] wrote: Domingos Jr. wrote: Carlos Gomes wrote: Claro que não

Re: [obm-l] equaçao BIquadrada

2005-08-02 Por tôpico Bernardo Freitas Paulo da Costa
acha as raizes quadradas pra teras duas solucoes. Espero que tenha ajudado,Abracos,-- Bernardo Freitas Paulo da Costa On 8/2/05, elton francisco ferreira [EMAIL PROTECTED] wrote: NA RESOLUÇAO DA EQUAÇAO NA VARIÁVEL X, PARA U = IR - {-1, 1}: a^2 + 4/ x^2 + 1 = 4 - a^2/ + a^2 + 1 + 4x^4/ x^4 – 1

[obm-l] Re: Espaço Vetorial

2005-08-04 Por tôpico Bernardo Freitas Paulo da Costa
, On 8/4/05, Denisson [EMAIL PROTECTED] wrote: Como se prova que todo espaço vetorial possui uma base? Obrigado -- Denisson -- Bernardo Freitas Paulo da Costa = Instruções para entrar na lista, sair da lista e usar

Re: [obm-l] Subconjunto fechado e denso em R

2005-08-12 Por tôpico Bernardo Freitas Paulo da Costa
certo, -- Bernardo Freitas Paulo da Costa On 8/12/05, Ana Evans Merryl [EMAIL PROTECTED] wrote: Oi Algúem poderia dar um exemplo de um subconjunto proprio de R que seja fechado e denso em R? O único exemplo que achei de conjunto fechado e denso em R é o próprio R. Obrigada Ana

Re: RES: [obm-l] conjunto de irracionais fechado com relacao aa soma

2005-08-12 Por tôpico Bernardo Freitas Paulo da Costa
desta série é um racional, calcule a diferença (use que a(k) é limitada para isso) e veja que ela é muito menor do que o maior denominador (estime a potência p tal que | irr - truncamento_em_n | = (X^(-n!))^p) Eu acho que é por aí. Abraços, -- Bernardo Freitas Paulo da Costa On 8/12/05, Artur Costa

Re: RES: [obm-l] Subconjunto fechado e denso em R

2005-08-13 Por tôpico Bernardo Freitas Paulo da Costa
O fecho de Q é R, pelo menos na topologia usual. E o de (R - Q) também. -- Bernardo Freitas Paulo da Costa On 8/13/05, fabiodjalma [EMAIL PROTECTED] wrote: Qual o fecho de Q? Em (17:33:44), obm-l@mat.puc-rio.br escreveu: Na realidade, voce provou que, em todo espaco topologico X, o

Re: [obm-l] solução natural

2005-08-13 Por tôpico Bernardo Freitas Paulo da Costa
Paulo da Costa On 8/13/05, Júnior [EMAIL PROTECTED] wrote: Prove que a equação 1/x^3 + 1/(x^2)y + 1/xy^2 + 1/y^3 =1 não possui solução natural. = Instruções para entrar na lista, sair da lista e usar a lista em http

Re: RES: [obm-l] conjunto de irracionais fechado com relacao aa soma

2005-08-14 Por tôpico Bernardo Freitas Paulo da Costa
algébrico de grau n (ou seja, existe um polinômio de grau n, com coeficientes inteiros, tal que \alpha é raiz deste polinômio), dá pra mostrar que, para todo p, q inteiros, q != 0, temos: |\alpha - p/q| 1/(q^n). Abraços, -- Bernardo Freitas Paulo da Costa On 8/14/05, Angelo Barone Netto [EMAIL

Re: [obm-l] convergencia da sequencia das derivadas

2005-08-18 Por tôpico Bernardo Freitas Paulo da Costa
eu concordo - ultrapassada em muito em utilidade pela de Lebesgue) que ainda assim tem um pouco de aplicação. Fiquei curioso: você pode dar detalhes desta suas g_n? Abraços, -- Bernardo Freitas Paulo da Costa On 8/18/05, Artur Costa Steiner [EMAIL PROTECTED] wrote: No meu caso, eu tenho um

Re: [obm-l] naturais

2005-08-25 Por tôpico Bernardo Freitas Paulo da Costa
, temos x == 1 ou x == -1, ent~ao x^2 == 1 das duas formas. Daí, a^2 == 1, e o mesmo vale para b^2 e c^2. Daí, temos 1 + 1 + 1 == 0 e a^2 + b^2 + c^2 == 0. Abraços, -- Bernardo Freitas Paulo da Costa On 8/25/05, Paulo Santa Rita [EMAIL PROTECTED] wrote: Ola Renato e demais colegas desta lista

Re: [obm-l] Derivada convexa

2005-08-25 Por tôpico Bernardo Freitas Paulo da Costa
= -- Bernardo Freitas Paulo da Costa = Instruções para entrar na lista, sair da lista e usar a lista em http://www.mat.puc-rio.br/~nicolau/olimp/obm-l.html =

Re: [obm-l] Probabilidade

2005-08-28 Por tôpico Bernardo Freitas Paulo da Costa
ficar. Ora, se D. sai, isso é 1/2; se J. sai, isso é 1. Logo, temos 1/2* 1/3 + 1/3 = 1/6 + 1/3 = 1/2. Assim, temos que P(D. sai | C. vai ficar) = 1/2 * 1/3 / (1/2) = 1/3, como antes. Abraços, -- Bernardo Freitas Paulo da Costa On 8/28/05, Luiz Viola [EMAIL PROTECTED] wrote: Parece simples...mas

Re: [obm-l] Provar que existem racionais que satisfazem.....

2005-09-01 Por tôpico Bernardo Freitas Paulo da Costa
Freitas Paulo da Costa On 9/1/05, alencar1980 [EMAIL PROTECTED] wrote: Pessoal, Será que alguém poderia me ajudar com este probleminha: Sejam a,b e x reais tais que: a+b x. Prove que existem r1 e r2 racionais tais que r1+r2x, ar1 e br2. O problema me pareceu bem intuitivo usando que

Re: [obm-l] dertivada como limite de uma seq. de funcoes continuas

2005-09-02 Por tôpico Bernardo Freitas Paulo da Costa
grande), n~ao teremos truncado em volta da bola de raio 1/3n em volta de x, o que diz que, a partir daí, SE A DERIVADA EM x EXISTIR, g_n(x) - f'(x). (na verdade, basta lateral à direita, que é o que estamos calculando) Acho que é isso. -- Bernardo Freitas Paulo da Costa On 9/2/05, Artur Costa

Re: [obm-l] dertivada como limite de uma seq. de funcoes continuas

2005-09-02 Por tôpico Bernardo Freitas Paulo da Costa
calculando) Acho que é isso. -- Bernardo Freitas Paulo da Costa On 9/2/05, Artur Costa Steiner [EMAIL PROTECTED] wrote: Eu estou tentando demonstrar a seguinte afirmacao, mas encontrei dificuldade em alguns casos particulares. Se f eh dervavel em um intervalo aberto I, entao f' eh dada pelo limite de

Re: [obm-l] O Problema do jipe

2005-09-03 Por tôpico Bernardo Freitas Paulo da Costa
vai (bom, depois você generalisa, pra fazer as contas) Fui, -- Bernardo Freitas Paulo da Costa On 9/3/05, Alamir Rodrigues [EMAIL PROTECTED] wrote: Se trata de um famoso enigma logístico da Segunda Guerra Mundial. Em essência, esse problema pede que você cruze o deserto do Saara, com 3200

[obm-l] Revis~ao final dos mails

2005-09-03 Por tôpico Bernardo Freitas Paulo da Costa
) Beijos, -- Bernardo Freitas Paulo da Costa = Instruções para entrar na lista, sair da lista e usar a lista em http://www.mat.puc-rio.br/~nicolau/olimp/obm-l.html =

Re: [obm-l] Livros da MIR

2005-09-04 Por tôpico Bernardo Freitas Paulo da Costa
Cara, o site mudou de nome (tinha uma pagina velha que ainda tinha isso, a unica no Google, e que disse que isso era a Livraria de Fisica da Usp), agora tem um i a mais: http://www.livifusp.com.br/. Espero que seja este mesmo. Abraços, -- Bernardo Freitas Paulo da Costa On 9/4/05, Fernando

Re: [obm-l] problema - fatoracao

2005-09-05 Por tôpico Bernardo Freitas Paulo da Costa
Bom, eu n~ao fiz as contas, mas acho que pode haver soluçoes com +-1 e +-2005 também, que s~ao fatoraç~oes aceitáveis de 2005! Abraços, -- Bernardo Freitas Paulo da Costa On 9/5/05, Renato Lira [EMAIL PROTECTED] wrote: perfeitamente. voce so esqueceu das sequintes possibilidades: {(y-x

Re: [obm-l] Um bom limite

2005-09-13 Por tôpico Bernardo Freitas Paulo da Costa
eu fiz uma vez) Abraços, -- Bernardo Freitas Paulo da Costa On 9/13/05, [EMAIL PROTECTED] [EMAIL PROTECTED] wrote: Também chegou às minhas mãos um limite. Quer se saber se ele o limite existe ou não. Caso exista determiná-lo. Aí vai: lim (x^2+y^2)*e^1/(x^2+y^2) (x,y)-(0,0

Re: [obm-l] desigualdade

2005-09-13 Por tôpico Bernardo Freitas Paulo da Costa
procede a demonstração: Mostre que ln 2 (2/5)^(2/5). log 2 (na base 2) = 1 ln 2 evidente 1 ln 2 (2/5)^(2/5) 1 (2/5)^(2/5) 5^(2/5) 2^(2/5) . FIM. Júnior. -- Bernardo Freitas Paulo da Costa

Re: [obm-l] desigualdade

2005-09-14 Por tôpico Bernardo Freitas Paulo da Costa
) é mais ou menos (1/2)^(1/2) ~= 0.707, daí tem que ver com mais cuidado. -- Bernardo Freitas Paulo da Costa On 9/14/05, Fernando Aires [EMAIL PROTECTED] wrote: Caros, On 08/09/05, Júnior [EMAIL PROTECTED] wrote: Preciso de ajuda nesse probleminha: Sem usar tábua de log ou uma

Re: [obm-l] NOVA trigonometria?

2005-09-18 Por tôpico Bernardo Freitas Paulo da Costa
se prestam para tais e tais calculos que os fisicos faziam, mas ainda estavam sem uma formalização. Isso é uma caracteristica importante. Bom, valeu pela divulgação, isso também faz parte da vida matematica! -- Bernardo Freitas Paulo da Costa On 9/17/05, Fabio Niski [EMAIL PROTECTED] wrote: Um

Re: [obm-l] RES: [obm-l] obt enção de números irracionais

2005-09-19 Por tôpico Bernardo Freitas Paulo da Costa
caracterizaç~ao geral que exclua estes casos triviais? (exceto dizer L.I. sobre os racionais, que é exatamente o que eu falei) Abraços, -- Bernardo Freitas Paulo da Costa On 9/19/05, Artur Costa Steiner [EMAIL PROTECTED] wrote: Na realidade, se n 1 e m1 sao numeros inteiros e n^(1/m) nao for inteiro

Re: [obm-l] A LINGUAGEM DA CIÊNCIA!

2005-10-07 Por tôpico Bernardo Freitas Paulo da Costa
, da pra ser vista como uma projeção 1d (que é bem mais facil) o que leva a soluções mais elegantes. E ainda falta a idéia de que o momento angular REALMENTE mede esta inércia... que muito poucas vezes é mencionado. Abraços, -- Bernardo Freitas Paulo da Costa On 10/7/05, Claudio Buffara [EMAIL

Re: [obm-l] Caracteres matemáticos

2005-10-10 Por tôpico Bernardo Freitas Paulo da Costa
cara fez o que achava melhor, mas nao ficou tao bom assim... Mas vamos esperar o veredito final. -- Bernardo Freitas Paulo da Costa On 10/10/05, Maurício [EMAIL PROTECTED] wrote: Oi, pessoal, Eu estava dando uma olhada nas tabelas do Unicode (www.unicode.org) e vi que elas possuem um

Re: [obm-l] sequencia sem subseq. convergentes

2005-10-10 Por tôpico Bernardo Freitas Paulo da Costa
detalhe. Abraços, -- Bernardo Freitas Paulo da Costa On 10/10/05, Artur Costa Steiner [EMAIL PROTECTED] wrote: Este problema eh interessante, e a unica prova que conheco nao eh muito evidente. Talvez haja uma solucao mais simples: Mostre que a sequencia de funcoes (sen(n*x)), n=1,2,3., x em

Re: [obm-l] sequencia sem subseq. convergentes

2005-10-11 Por tôpico Bernardo Freitas Paulo da Costa
para p finito Conv em Medida exponencialmente rápida = Conv qtp Bom, agora a referência (para demonstraç~oes e uma figurinha bem bonita) Curso de Teoria da Medida, A. Armando de Castro Jr, Projeto Euclides / IMPA, pag 103 e 104 Até mais, -- Bernardo Freitas Paulo da Costa On 10/10/05, Artur

Re: [obm-l] Conjunto aberto e denso com medida eps

2005-10-11 Por tôpico Bernardo Freitas Paulo da Costa
conjuntos. Dá pra provar isso usando a definiç~ao do Arthur, mas isso eu chamaria de um outro exercício porque é bem usado em outras coisas. Abraços, -- Bernardo Freitas Paulo da Costa On 10/11/05, Artur Costa Steiner [EMAIL PROTECTED] wrote: Boa tarde, Eu acho este problema interessante

Re: [obm-l] funções

2005-10-20 Por tôpico Bernardo Freitas Paulo da Costa
Escreva f(x) = ( f(x) + f(-x) )/2 + ( f(x) - f(-x) )/2, repare que os termos s~ao, respectivamente, par e ímpar. Abraços -- Bernardo Freitas Paulo da Costa On 10/20/05, Eder Albuquerque [EMAIL PROTECTED] wrote: Olá, Pessoal, essa é velha, mas não tô lembrando como fazer... A questão é

Re: [obm-l] m^x + x (off-topic)

2005-10-31 Por tôpico Bernardo Freitas Paulo da Costa
Só uma idéia (nem testei ainda) m^x tem período que divide phi(n) (é isso mesmo?), enquanto x tem período n. Agora, eu acho que phi(n) e n s~ao primos entre si. Se for, acho que acabou Abraços -- Bernardo Freitas Paulo da Costa On 10/31/05, claudio.buffara [EMAIL PROTECTED] wrote: Desculpem o

Re: [obm-l] Limite superior para a soma de logaritmos

2005-11-07 Por tôpico Bernardo Freitas Paulo da Costa
pelo sinal ~. Melhores aproximaç~oes para a soma (por integrais melhores) dao a fórmula com mais precisao. -- Bernardo Freitas Paulo da Costa On 11/6/05, Denisson [EMAIL PROTECTED] wrote: Boa noite pessoal, Tou tentando encontrar uma função que limite superiormente a soma log1 + log2

Re: [obm-l] Matemática Discreta

2005-11-15 Por tôpico Bernardo Freitas Paulo da Costa
Sim, a diagonal (xRy se e só se x = y) e qualquer sub-relaç~ao dela (Exercício: prove que estas s~ao as únicas) -- Bernardo Freitas Paulo da Costa On 11/10/05, Tiago [EMAIL PROTECTED] wrote: Pode haver uma relação que seja simétrica e anti-simétrica ao mesmo tempo

Re: [obm-l] probabilidade (ufrj)

2005-11-28 Por tôpico Bernardo Freitas Paulo da Costa
). Esse é um bom exemplo em que mudar completamente o espaço de resultados possíveis simplifica bastante a resoluç~ao Se você quiser agora fazer com 10 moedas e 5 caras, vai ser um pouco mais complicado, mas no fim das constas dá no mesmo :) Abraços, -- Bernardo Freitas Paulo da Costa On 11/28/05

  1   2   3   4   5   6   7   >